Difference between revisions of "2019 AMC 12B Problems/Problem 22"

(Problem: Added problem statement.)
m (Problem: Added the set of solution)
Line 7: Line 7:
  
 
In which of the following intervals does <math>m</math> lie?
 
In which of the following intervals does <math>m</math> lie?
 +
 +
<math>\textbf{(A) }[9,26]\qquad\textbf{(B) }[27,80]\qquad\textbf{(C) }[81,242]\qquad\textbf{(D) }[243,728]\qquad\textbf{(E) }[729,\infty)</math>
  
 
==Solution==
 
==Solution==

Revision as of 17:39, 14 February 2019

Problem

Define a sequence recursively by $x_0 = 5$ and

$x_{n+1} = \frac{x_n^2 + 5x_n + 4}{x_n + 6}$

for all nonnegative integers $n$. Let $m$ be the least positive integer such that $x_m \leq 4 + \frac{1}{2^{20}}$.

In which of the following intervals does $m$ lie?

$\textbf{(A) }[9,26]\qquad\textbf{(B) }[27,80]\qquad\textbf{(C) }[81,242]\qquad\textbf{(D) }[243,728]\qquad\textbf{(E) }[729,\infty)$

Solution

See Also

2019 AMC 12B (ProblemsAnswer KeyResources)
Preceded by
Problem 21
Followed by
Problem 23
1 2 3 4 5 6 7 8 9 10 11 12 13 14 15 16 17 18 19 20 21 22 23 24 25
All AMC 12 Problems and Solutions